Administración     

Olimpiadas de Matemáticas
Página de preparación y problemas

OME Local
OME Nacional
OIM
OME Andalucía
Retos UJA
Selector
La base de datos contiene 1154 problemas y 775 soluciones.

LVI Olimpiada Matemática Española (fase nacional) — 2020

Sesión 1 —  Online (Almería), martes 14 de julio de 2020

Problema 1100
Decimos que un polinomio $p(x)$ con coeficientes reales es almeriense si tiene la forma $p(x)=x^3+ax^2+bx+a$ y sus tres raíces son números reales positivos en progresión aritmética. Halla todos los polinomios almerienses tales que $p(\frac{7}{4})=0$.
pistasolución 1info
Pista. Plantea las ecuaciones de Cardano-Vieta suponiendo que las tres raíces están en progresión aritmética.
Solución. Sean $0\lt\alpha\leq\beta\leq\gamma$ las tres raíces. Por tanto \begin{align*} p(x)&=x^3+ax^2+bx+a=(x-\alpha)(x-\beta)(x-\gamma)\\ &=x^3-(\alpha+\beta+\gamma)x^2+(\alpha\beta+\beta\gamma+\gamma\alpha)x-\alpha\beta\gamma. \end{align*} Igualando coeficientes obtenemos las relaciones de Cardano-Vieta: \[\alpha+\beta+\gamma=-a,\qquad \alpha\beta+\beta\gamma+\gamma\alpha=b,\qquad \alpha\beta\gamma=-a.\] Ahora bien, como los números están en progresión aritmética, podemos escribir $\alpha=\beta-d$ y $\gamma=\beta+d$, siendo $d\geq 0$. Sustituyendo estos valores, las ecuaciones de Cardano-Vieta se pueden reescribir de la siguiente manera: \[3\beta=-a,\qquad 3\beta^2-d^2=b,\qquad \beta(\beta^2-d^2)=-a.\] De la primera y la tercera obtentemos que $\beta^2-d^2=3$. Para terminar, distinguiremos casos dependiendo de cuál de las raíces es igual a $\frac{7}{4}$, información que aún no hemos usado.
  • Si $\beta+d=\frac{7}{4}$, entonces la ecuación $3=\beta^2-d^2=(\beta+d)(\beta-d)$ nos dice que $\beta-d=\frac{12}{7}$. Sumando y restando a esta última la ecuación $\beta-d=\frac{7}{4}$, llegamos fácilmente a que $\beta=\frac{97}{56}$ y $d=\frac{1}{56}$. Usando ahora las otras ecuaciones, se tiene que $a=-3\beta=\frac{-291}{56}$ y $b=3\beta^2-d^2=\frac{14113}{1568}$.
  • Si $\beta=\frac{7}{4}$, entonces $d^2=\beta^2-3=\frac{1}{16}$, luego $d=\frac{1}{4}$ ya que habíamos supuesto que $d\geq 0$. Por lo tanto, $a=-3\beta=\frac{-21}{4}$ y $b=3\beta^2-d^2=\frac{73}{8}$.
  • Si $\beta-d=\frac{7}{4}$, entonces la ecuación $3=\beta^2-d^2=(\beta+d)(\beta-d)$ nos dice que $\beta+d=\frac{12}{7}$, de donde $d=\frac{-1}{56}$. Esto no nos da ninguna solución ya que habíamos supuesto que $d\geq 0$.

En resumen, hay dos polinomios almerienses que tienen a $\frac{7}{4}$ por raíz: \[p_1(x)=x^3-\frac{291}{56}x^2+\frac{14113}{1568}x-\frac{291}{56}\qquad\text{y}\qquad p_2(x)=x^3-\frac{21}{4}x^2+\frac{73}{8}x-\frac{21}{4}.\]

Si crees que el enunciado contiene un error o imprecisión o bien crees que la información sobre la procedencia del problema es incorrecta, puedes notificarlo usando los siguientes botones:
Informar de error en enunciado Informar de procedencia del problema
Problema 1101
Consideramos la sucesión de números enteros $\{f(n)\}_{n=1}^\infty$ definida por:
  • $f(1) = 1$.
  • Si $n$ es par, $f(n) = f(n/2)$.
  • Si $n\gt 1$ es impar y $f(n-1)$ es impar, entonces $f(n)=f(n-1)-1$.
  • Si $n\gt 1$ es impar y $f(n-1)$ es par, entonces $f(n)=f(n-1)+1$.
  1. Calcular $f(2^{2020}-1)$.
  2. Demostrar que $\{f(n)\}_{n=1}^\infty$ no es periódica, es decir, no existen enteros positivos $t$ y $n_0$ tales que $f(n+t)=f(n)$ para cualquier $n\geq n_0$.
Sin pistas
Sin soluciones
info
Si crees que el enunciado contiene un error o imprecisión o bien crees que la información sobre la procedencia del problema es incorrecta, puedes notificarlo usando los siguientes botones:
Informar de error en enunciado Informar de procedencia del problema
Problema 1102
A cada punto $(x,y,z)$ del espacio con coordenadas enteras, le asignamos un color de entre $p$ colores posibles. Demostrar que forzosamente existe algún ortoedro cuyos vértices tienen coordenadas enteras y son todos del mismo color.
Sin pistas
Sin soluciones
info
Si crees que el enunciado contiene un error o imprecisión o bien crees que la información sobre la procedencia del problema es incorrecta, puedes notificarlo usando los siguientes botones:
Informar de error en enunciado Informar de procedencia del problema

Sesión 2 —  Online (Almería), miércoles 15 de julio de 2020

Problema 1103
Ana y Benito juegan a un juego que consta de $2020$ rondas. Inicialmente, en la mesa hay $2020$ cartas numeradas de $1$ a $2020$ y Ana tiene una carta adicional con el número $0$. En la ronda $k$-ésima, el jugador que no tiene la carta $k-1$ decide si toma la carta $k$ o si se la entrega al otro jugador. El número de cada carta indica su valor en puntos. Al terminar el juego, gana quien tiene más puntos. Determinar qué jugador tiene estrategia ganadora o si ambos jugadores pueden forzar el empate. En cualquier caso, describir la estrategia a seguir.
Sin pistas
Sin soluciones
info
Si crees que el enunciado contiene un error o imprecisión o bien crees que la información sobre la procedencia del problema es incorrecta, puedes notificarlo usando los siguientes botones:
Informar de error en enunciado Informar de procedencia del problema
Problema 1104
En un triángulo acutángulo $ABC$, sea $M$ el punto medio del lado $AB$ y $P$ el pie de la altura sobre el lado $BC$. Prueba que si $AC+BC=\sqrt{2}AB$, entonces la circunferencia circunscrita del triángulo $BMP$ es tangente al lado $AC$.
Sin pistas
Sin soluciones
info
Si crees que el enunciado contiene un error o imprecisión o bien crees que la información sobre la procedencia del problema es incorrecta, puedes notificarlo usando los siguientes botones:
Informar de error en enunciado Informar de procedencia del problema
Problema 1105
Sea $S$ un subconjunto finito de los números enteros. Definimos $d_2(S)$ y $d_3(S)$ de la siguiente manera:
  • $d_2(S)$ es el número de elementos $a\in S$ para los que existen $x,y\in\mathbb{Z}$ tales que $x^2-y^2=a$.
  • $d_3(S)$ es el número de elementos $a\in S$ para los que existen $x,y\in\mathbb{Z}$ tales que $x^3-y^3=a$.
  1. Sea $m$ un número entero y sea $S=\{m,m+1,\ldots, m+2019\}$. Probar que \[d_2(S)\gt \frac{13}{7}d_3(S).\]
  2. Sea $n$ un número entero positivo y sea $S_n=\{1,2,\ldots,n\}$. Probar que existe un número $N$ de manera que, si $n\gt N$, entonces \[d_2(S_n)\gt 4 d_3(S_n).\]
Sin pistas
Sin soluciones
info
Si crees que el enunciado contiene un error o imprecisión o bien crees que la información sobre la procedencia del problema es incorrecta, puedes notificarlo usando los siguientes botones:
Informar de error en enunciado Informar de procedencia del problema
José Miguel Manzano © 2010-2024. Esta página ha sido creada mediante software libre